Neurology Flashcards

1
Q

Describe neurofibromatosis, it’s types, and features and diagnosis.

A

There are two types NF1 and NF2 both are automsomal dominant.

NF1 features with cafe-au-lait spots, axillary/groin freckles, peripheral neurofibromas, iris hamartomas, scoliosis, phaeochromocytomas.

NF2 features with bilateral acoustic neuromas.

Diagnosis of NF1 assessed by seven clinical criteria at least two of which should be present:

  • Six or more Cafe-au-lait spots or hyper pigmented macules greater than of equal to 5mm in diameters in pre-pubertal individuals and 15mm in post-pubertal individuals.
  • Two or more neurofibromas or one plexiform neurofibroma
  • Axillary of inguinal freckles
  • Two or more iris hamartomas (Lisch nodules), often identified only thought slit-lamp examination by an ophthalmologist
  • optic nerve glioma
  • Sphenoid dysplasia or typical long-bone abnormalities such as psuedouarthrosis
  • First-degree relative with NF1.
How well did you know this?
1
Not at all
2
3
4
5
Perfectly
2
Q

What are the features of a UMN lesion?

A
Affects muscle groups
Upgoing plantar reflex (+ve babinski)/clonus
Hyperreflexia
No fasciculations
Spasticity/hypertonia
How well did you know this?
1
Not at all
2
3
4
5
Perfectly
3
Q

What are the featured of a LMN lesion?

A

Caused by damage from anterior horn cells, nerve roots, plexi, or peripheral nerves

Wasting and fasciculations
Hyporeflexia
Downward going plantars (-ve babinski)
Hypotonia (flaccidity)

How well did you know this?
1
Not at all
2
3
4
5
Perfectly
4
Q

What does the anterior cerebral artery supply and what symptoms arise from its occlusion?

A

Supplies the frontal and medial part of the cerebrum.

Occlusion may cause a weak,numb contralateral leg +/- similar but often milder picture in the arms. The face is spared.

How well did you know this?
1
Not at all
2
3
4
5
Perfectly
5
Q

What does the middle cerebral artery supply and what symptoms arise from its occlusion?

A

Supplies the lateral part of each hemisphere

Occlusion may cause a weak contralateral hemiparesis, Hemisensory loss (esp arm + face), contralateral homonymous hemianopia.

How well did you know this?
1
Not at all
2
3
4
5
Perfectly
6
Q

What does the posterior cerebral artery supply and what symptoms arise from its occlusion?

A

Supplies occipital lobe

Occlusion gives contralateral homonymous hemianopia (often with macular sparing)

How well did you know this?
1
Not at all
2
3
4
5
Perfectly
7
Q

Describe subclavian steal syndrome

A

Subclavian artery stenosis proximal to the origin of the vertebral artery may cause blood to be stolen by retrograde flow down this vertebral artery down into the arm, causing brain stem ischaemia typically after use of the arm.

Suspect if the BP in each arm differs by >20mmHg

How well did you know this?
1
Not at all
2
3
4
5
Perfectly
8
Q

What are some causes of an acute single episode headache?

A
  • With meningism: meningitis, encephalitis, subarachnoid haemorrhage
  • Head injury
  • Venous sinus thrombosis
  • Sinusitis
  • Tropical illness
  • Low pressure headache
  • Acute glaucoma
How well did you know this?
1
Not at all
2
3
4
5
Perfectly
9
Q

What are some causes of recurrent acute attacks of headache?

A
  • migraine
  • cluster headache
  • trigeminal neuralgia
  • recurrent (mollaret’s) meningitis
How well did you know this?
1
Not at all
2
3
4
5
Perfectly
10
Q

What are some causes of subacute onset headaches?

A

Giant cell arteritis should be excluded in all >50yrs old with a headache that has lasted a few weeks.

How well did you know this?
1
Not at all
2
3
4
5
Perfectly
11
Q

What are some causes of chronic headache?

A
  • Tension Headache
  • raised ICP
  • Medication overuse (analgesic rebound) headache
How well did you know this?
1
Not at all
2
3
4
5
Perfectly
12
Q

Describe tension headache, its symptoms, and management

A

Symptoms:
Usual cause of bilateral, non pulsating headache +/- scalp tenderness, but without vomiting or sensitivity to head movement.

Management:

  • Stress relief, massage, or antidepressant (low dose amitryptilline) may help.
  • Consider seeing optician
How well did you know this?
1
Not at all
2
3
4
5
Perfectly
13
Q

Describe raised intracranial pressure, its symptoms and tests.

A

Symptoms:
Typically worse on waking, lying, bending forward, or coughing. Also vomiting, papilloedema, seizure, false-localising signs, or odd behaviour

Tests: Do imaging to exclude a space-occupying lesion, and consider idiopathic intracranial hypertension. LP is contraindicated until after imaging!

How well did you know this?
1
Not at all
2
3
4
5
Perfectly
14
Q

Describe Acute Glaucoma, it’s presentation, symptoms, and management.

A

Presentation: Typically elderly, long-sighted people. Constant, aching pain develops rapidly around one eye, radiating to the forehead. Attacks may be precipitated by dilating eye-drops, emotional upset or sitting in the dark.

Symptoms: markedly reduced vision, visual haloes, nausea/vomiting.

Signs: red congested eye, cloudy cornea, fixed dilated non-responsive pupil (may be oval), decreased acuity.

Management:
seek expert help, if delay of treatment of >1h start acetazolamide 500mg IV over several minutes.

How well did you know this?
1
Not at all
2
3
4
5
Perfectly
15
Q

Describe Cluster Headaches, its features, and management

A

Unilateral severe headache that occurs once or twice a day for 15-180mins and is often nocturnal, usually lasting several weeks, with the clusters themselves occuring typically once a year. Clusters may be episodic or chronic. 5 times more common in men, can occur at any age and more common in smokers.

Features:
-Rapid-onset of excruciating stabbing pain around one eye that may become watery and bloodshot with lid swelling, lacrimation, facial flushing, rhinorrhoea, miosis+/- ptosis.

Management:

  • acute attack: 100% O2 (80% response rate within 15 minutes) + Sumatriptan SC 6mg (75% response rate within 15 minutes)
  • Prophylaxis: Verapamil is the drug of choice, some evidence supports a tapering dose of prednisolone
  • Neurology referral
How well did you know this?
1
Not at all
2
3
4
5
Perfectly
16
Q

Describe epilepsy and its suggestive features.

A

Epilepsy is a recurrent tendency to spontaneous, intermittent, abnormal electrical activity in part of the brain, manifesting as seizures.

Features suggestive of epilepsy: attacks when asleep or lying down, aura, identifiable triggers (e.g. tv) altered breathing, cyanosis, typical tonic-clonic movements, incontinence, tongue-biting, prolonged post-ictal drowsiness (slow recovery), confusion, amnesia, and transient focal paralysis (Todd’s palsy)

How well did you know this?
1
Not at all
2
3
4
5
Perfectly
17
Q

What are some causes of seizures?

A
  • Idiopathic ~ 2/3
  • Structural: (cortical scaring, developlmental, space-occupying lesion, stroke, hippocampal sclerosis, vascular malformations)
  • Others: tuberous sclerosis, sarcoidosis, SLE, PAN
  • Non-epileptic causes: trauma, stroke, haemorrhage, increased ICP, alcohol or benzodiazepine withdrawal, metabolic disturbance (e.g. hypoxia, hypo/hypernatraemia, hypocalcaemia, hypo/hyperglycaemia, uraemia) liver disease, infection, hyperthermia, drugs (tricyclics, cocaine, tramadol, theophylline), pseudoseizures.
How well did you know this?
1
Not at all
2
3
4
5
Perfectly
18
Q

What are the classifications of seizures?

A

The basic classification of epilepsy has changed in recent years. The new basic seizure classification is based on 3 key features:

  • where seizure begin in the brain
  • level of awareness during a seizure
  • other features of seizures

Focal seizures:

  • previously termed partial seizures
  • these start in specific area, on one side of the Brain.
  • the level of awareness can vary, the terms focal aware (previously simple partial) or focal awareness impaired (previously complex partial) and awareness unknown are used to further differentiate focal seizures
  • Further to this focal seizures can be classified as being motor (e.g. Jacksonian march), non-motor ( e.g. dejavu, jamais vu). or having other features such as aura.

Generalised seizures:

  • these engage or involve networks job both sides of the brain at onset.
  • Consciousness is lost immediately.
  • Generalised seizures can be further subdivided into motor (e.g. tonic-clonic) and non-motor (e.g. absence)
  • specific types include, tonic-clonic, tonic, clonic, absence, myoclonic, atonic.

Unknown onset: a term reserved for when origin of the seizure is unknown.

Focal to bilateral:

  • starts on one side in a specific area before spreading to both lobes
  • previously termed secondary generalised seizures.
How well did you know this?
1
Not at all
2
3
4
5
Perfectly
19
Q

Define a focal aware seizure

A

Awareness is unimpaired. with focal motor, sensory, autonomic or psychic symptoms. no post-ictal symptoms.

How well did you know this?
1
Not at all
2
3
4
5
Perfectly
20
Q

Define a focal impaired awareness seizure

A

awareness is impaired. may have a simple partial onset or impaired awareness at onset. most commonly arise from the temporal lobe. Post-ictal confusion is common with seizure arising from the temporal lobe, whereas recovery is rapid after seizures in the frontal lobe.

How well did you know this?
1
Not at all
2
3
4
5
Perfectly
21
Q

Define a focal to bilateral seizure

A

occurs in ~ 2/3 of patients with partial seziures, electrical disturbances begin focally as either complex or simple seizures but then spread widely, causing a secondary generalised seizure which is typically convulsive.

How well did you know this?
1
Not at all
2
3
4
5
Perfectly
22
Q

Describe Absence seizures, and its management

A

Brief (less than 10s) pauses, e,g, suddenly stops talking mid-sentence then continues where they left off. Presents in childhood 4-8yrs.

Management:

  • Sodium Valproate, Ethosuximide
  • Good prognosis 90-95% become seizure free in adolescence.
How well did you know this?
1
Not at all
2
3
4
5
Perfectly
23
Q

Define tonic-clonic seizures

A

loss of consciousness. limbs stiffen (tonic), the jerk (clonic) may have one without the other. post-ictal confusions and drowsiness occur.

How well did you know this?
1
Not at all
2
3
4
5
Perfectly
24
Q

Define myoclonic seizures

A

sudden jerk of a limb, face of trunk. the patient may be thrown suddenly to the ground or have a violently disobedient limb.

How well did you know this?
1
Not at all
2
3
4
5
Perfectly
25
Q

Describe atonic (akinetic) seizures

A

sudden loss of muscle tone causing a fall, no loss of consciousness

How well did you know this?
1
Not at all
2
3
4
5
Perfectly
26
Q

What are some localising features of partial focal temporal lobe seizures?

A
  • Automatisms (brief complex motor phenoma with impaired awareness/ no recollection e.g. may be oral or manual.
  • abdominal rising sensation +/- pain
  • dysphasia
  • memory phenonema e.g. dejavu or the opposite jamais vu
  • hippocampal involvement may cause emotional disturbance e.g. terror, anger
  • Uncal involvement may cause hallucinations fo smell or taste
  • Auditory hallucinations
  • delusional behaviour
How well did you know this?
1
Not at all
2
3
4
5
Perfectly
27
Q

What are some localising features of partial focal frontal lobe seizures?

A
  • Motor features
  • motor arrest
  • subtle behavioural distubances
  • dysphasia or speech arrest
  • post-ictal todd’s palsy
How well did you know this?
1
Not at all
2
3
4
5
Perfectly
28
Q

What are some localising features of partial focal parietal lobe seizures?

A
  • sensory disturbances

- motor symptoms

How well did you know this?
1
Not at all
2
3
4
5
Perfectly
29
Q

What are some localising features of partial focal occipital lobe seizures?

A

visual phenomena such as spots lines or flashes

How well did you know this?
1
Not at all
2
3
4
5
Perfectly
30
Q

Define pseudo- or psychogenic seizures

A

these are not infrequent suspect if there are uncontrollable symptoms, no learning disabilities, and CNS, CT, MRI and EEG are all normal. May be part of Munchausen’s syndrome

How well did you know this?
1
Not at all
2
3
4
5
Perfectly
31
Q

What are the drug used in treatment of epilepsy?

A

Generalised tonic-clonic seizures - sodium valproate or lamotrigine are 1st line, then carbamazepine or topiramate.

Absence seizures - ethosuximide, sodium valproate, lamotrigine.

Tonic, atonic, and myoclonic seizures same as tonic-clonic but no carbamazepine as it may worsen seizures.

Focal seizures - carbamazepine is first line then lamotrigine and valproate

How well did you know this?
1
Not at all
2
3
4
5
Perfectly
32
Q

What are the side effects of sodium valproate

A
V alproate side effects
A ppetite increased leading to weight gain
L iver failure (LFT monitoring)
P ancreatitis
R eversible hair loss (grows back curly)
O edema
A taxia
T eratogenicity, tremor, thrombocytopenia (Monitor bloods)
E ncephalopathy
How well did you know this?
1
Not at all
2
3
4
5
Perfectly
33
Q

How do you assess risk of stroke in patients presenting with TIA?

A

ABCD2 score - >6 strongly predicts stoke, >4 should be assed within 24 hours all suspected TIA should be seen within 7 days.

A ge>60 = 1 point
B lood pressure >140/90 = 1 point
C linical features
    unilateral weakness = 2 points
    speech disturbance without weakness = 1 point
D uration of symptoms
    lasting > 1h = 2 points
    lasting 10-59 mins = 1 point
D iabetes = 1 point
How well did you know this?
1
Not at all
2
3
4
5
Perfectly
34
Q

Describe Cauda equina syndrome and its symptoms.

A

A neurosurgical emergency!

Symptoms: Features include back pain and radicular pain down the legs, assymetrical (alternating or bilateral), atrophic, areflexic paralysis of the legs. Sensory loss in a root distribution (saddle anaesthesia) and decreased sphincter tone causing bladder and bowel incontinence (Do PR)

How well did you know this?
1
Not at all
2
3
4
5
Perfectly
35
Q

What would you expect to see on CSF analysis from LP of suspected bacterial meningitis?

A

Appearance: cloudy
Glucose: low less than half plasma glucose
Protein: high >1g/L
White cells: 10-5000 polymorphs/mm

How well did you know this?
1
Not at all
2
3
4
5
Perfectly
36
Q

What would you expect to see on CSF analysis from LP of suspected tuberculous meningitis?

A

Appearance: slightly cloudy, fibrin web
Glucose: low less than half plasma glucose
Protein: high >1g/L
White cells: 10-1000 lymphocytes/mm

How well did you know this?
1
Not at all
2
3
4
5
Perfectly
37
Q

What would you expect to see on CSF analysis from LP of suspected viral meningitis?

A

Appearance: clear/cloudy
Glucose: 60-80% plasma glucose
Protein: normal/raised
White cells: 15-1000 lymphocytes/mm

How well did you know this?
1
Not at all
2
3
4
5
Perfectly
38
Q

What are the main features of Parkinsonism?

A
  • Tremor (worst at rest, often ‘pill-rolling’)
  • bradykinesia (diagnostic feature)
  • rigidity/ increased tone (tremor + rigidity gives cogwheel rigidity felt during pronation and supination of arm)
How well did you know this?
1
Not at all
2
3
4
5
Perfectly
39
Q

What are some causes of Parkinsonism?

A
  • idiopathic Parkinson’s,
  • drug induced (anti-psychotics and anti emetics (domperidone does not cross the blood brain barrier)
  • Progressie supranuclear palsy
  • Multiple system atrophy
  • Dementia Pugilistica (secondary to trauma/boxing)
  • Wilson’s disease
  • Post-encephalitis
  • HIV
  • vascular
  • Toxins e.g. Carbon monoxide, MPTP
How well did you know this?
1
Not at all
2
3
4
5
Perfectly
40
Q

Describe idiopathic Parkinson’s disease, it’s presentation, and management..

A

Incurable progressive disease characterised by degeneration of neurones in the substantia Nigra pars compacta associated with lewy bodies which leads to a reduction in striatal-like dopamine

Presentation: bradykinesia + one or more of: resting tremor, postural instability, rigidity. Typically one side is worst often right side. Also micrographia, depression, decreased cognition (Lewy body dementia), slow festinating gait, simian posture,

Management: levodopa, dopamine agonists (ropinirole), anticholinergics, MAO-B inhibitors (selegiline), COMT inhibitors (entacapone)

How well did you know this?
1
Not at all
2
3
4
5
Perfectly
41
Q

What are some causes of nystagmus?

A
  • horizontal nystagmus may be due to vestibular or cerebellum lesions. If it occurs more in the eye abducting cause may be MS. If also deafness and tinnitus may be peripheral cause e.g. VIIIth nerve lesion
  • vertical nystagmus requires specialist review, upbeat nystagmus classically occurs in lesions of midbrain or base of 4th ventricle. Downbeat nystagmus in foramen magnum lesions.
How well did you know this?
1
Not at all
2
3
4
5
Perfectly
42
Q

What are the symptoms of a cerebellum lesion?

A
D ysdiadokokinesis and dysmetria 
A taxia
S lurred speech/stoccato speech 
H ypotonia 
I ntention tremor
N ystagmus 
G ait abnormality
How well did you know this?
1
Not at all
2
3
4
5
Perfectly
43
Q

Describe Rhomberg’s test and meaning of its results

A

Rhomberg’s test is used to assess a patient proprioception when standing with their eyes closed. A loss of balance is a positive test

In the context of ataxia a patient with a positive Rhomberg’s suggest the ataxia is sensory in nature if it is negative it suggest the ataxia is of cerebellum origin

How well did you know this?
1
Not at all
2
3
4
5
Perfectly
44
Q

Describe subarachnoid haemorrhage, its symptoms, and management, and complications

A

80% are due to rupture of saccular aneurysms. Common sites for berry aneurysms are the junction of posterior communicating artery with the internal carotid (may present with pupil changes indicating a IIIrd nerve palsy) or the junction of anterior communicating artery with the anterior cerebral artery or bifurcation of the middle cerebral artery.

Symptoms: Usually presents with sudden onset devastating occipital headache, vomiting, collapse,seizures and coma often follow. May report a sentinel headache. Signs include Kernigs sign, neck stiffness.

Management:

  • Do CT within 4 hours of presentation and if positive refer to neurosurgeons for endovascular coiling or less preferred clipping.
  • If CT negative and no contraindications do LP 12 hours after presentation. Xanthachromia confirms Subarachnoid haemorrhage.
  • Maintain BP and nimodipine to reduce vasospasm.

Complications:

  • Hydrocephalus (occurs a few days after)
  • Raised Intracranial pressure
  • Rebleeding (Occurs in first 1-2 days or after 2 weeks)
  • Delayed Ischaemia (Occurs around day 10)
How well did you know this?
1
Not at all
2
3
4
5
Perfectly
45
Q

What is Kernig’s sign?

A

Kernig’s sign is a positive when the hip is flexed to 90 degrees and there is paining resistance of extension of the knee. It is indicative of subarachnoid haemorrhage or meningitis

How well did you know this?
1
Not at all
2
3
4
5
Perfectly
46
Q

What is battle’s sign?

A

Bruising of the mastoid process a sign of basal skull fracture

How well did you know this?
1
Not at all
2
3
4
5
Perfectly
47
Q

What is Panda or Raccoon eyes a sign of?

A

It is bruising of the eyes a sign of basal skull fracture

How well did you know this?
1
Not at all
2
3
4
5
Perfectly
48
Q

Describe IIIrd Cranial Nerve palsy and its causes

A

Ptosis, large pupil, eye down and out.

If lesion present on it own causes may be:

  • Medical (pupil sparing) e.g. Diabetes, HTN, GCA, syphillis idiopathic.
  • Surgical (early pupil involvement) e.g. posterior communicating artery aneurysm, Increased ICP, tumours.
How well did you know this?
1
Not at all
2
3
4
5
Perfectly
49
Q

Describe IVth Cranial Nerve palsy and its causes.

A

Diplopia on looking down and in (often noticed on descending stairs) head tilting compensates for this (ocular torticollis).

Lesion rarely occurs on its own but possible after trauma to orbit. More commonly presents with other lesions.

How well did you know this?
1
Not at all
2
3
4
5
Perfectly
50
Q

Describe VIth Cranial Nerve palsy and possible causes.

A

The Abducens nerve. Horizontal diplopia on abduction.

False localising sign due to long intracranial course in Raised ICP, SOL,

How well did you know this?
1
Not at all
2
3
4
5
Perfectly
51
Q

Describe acoustic neuroma and its presentation, investigations, and management

A

It is a schwannoma arising from the vestibular nerve. They account for 80% of cerebello-pontine angle tumours. Commoner in females and also in neurofibromatosis.

Presentation: progressive unilateral tinnitus +/- sensorineural hearing loss, with vertigo occuring later. With progresion, ipsilateral Vth,VIIth, VIIIth, (IXth, Xth nerves may be affected later). Signs of increased ICP occur late, indicating a large tumour.

Investigations: MRI in all with unilateral tinnitus and deafness

Management: Surgery

How well did you know this?
1
Not at all
2
3
4
5
Perfectly
52
Q

Describe the red flags present in parkinson’s-plus syndrome

A
  • Early postural instability and vertical gaze palsy +/- falls, rigidity of trunk > in limbs, symmetrical onset, speech and swallowing problems, little tremor = Progressive Supranuclear Palsy (PSP)
  • Early autonomic features e.g. impotence/incontinence, postural BP drop, cerebellar + pyrimidal signs, rigidity>tremor = Multiple System Atrophy (MSA)
  • Fluctuating cognition with visual hallucinations and early dementia = lewy-body dementia
  • akinetic rigidity involving one limn and cortical sensory loss e.g. astereognosis (inability to recognise objects by touch alone) = cortico-basal degeneration
  • Pyramidal signs e.g. in a diabetic/hypertensive patient who falls or has gait problems with no festination = vascular parkinsonism
How well did you know this?
1
Not at all
2
3
4
5
Perfectly
53
Q

Describe the pull test

A

a sharp tug on the back of a patients shoulders to test for postural instability a positive result is one in which a patient is unable to correct there balance and continues falling backwards after the tug.

How well did you know this?
1
Not at all
2
3
4
5
Perfectly
54
Q

What are some causes of cerebellar syndrome?

A
Posterior Fossa Tumour
Alcohol
Sclerosis (Multiple)
Trauma
Rare e.g. Iithium
Inherited - Friedrich's ataxia
Epilepsy medications e.g. Phenytoin, carbamazepine
Stroke
How well did you know this?
1
Not at all
2
3
4
5
Perfectly
55
Q

Describe hemispatial neglect (hemiagnosia)

A

Damage to one hemisphere of the brain leads to a deficit in attention to and awareness of one side of observed space. It is usually contralateral to the lesion. Usually due to a lesion of the parietal cortex but in particular neglect is closely related to damage of the temporo-parietal junction and posterior parietal cortex.

How well did you know this?
1
Not at all
2
3
4
5
Perfectly
56
Q

Describe Visual extinction

A

Due to damage to the parietal lobe, similar but distinct from hemispatial neglect (hemiagnosia). Visual extinction describes the symptom in which patients have difficulty perceive contralesional stimuli e.g. moving fingers, when presented simultaneously wth an ipsilesional stimulus; but able to identify both stimuli when presented individually.

How well did you know this?
1
Not at all
2
3
4
5
Perfectly
57
Q

What is the difference between spasticity and rigidity

A

Spasticity is velocity dependant the initial fast movement is resisted and then gives way, usually uni-directional whereas rigidity is independent of velocity and is bidirectional.

How well did you know this?
1
Not at all
2
3
4
5
Perfectly
58
Q

Describe lateral medullary syndrome

A

Occurs after occlusion of most commonly the vertebral artery but also posterior inferior cerebellar artery.

Present with:

  • cerebellar features e.g. ataxia, nystagmus
  • brainstem features e.g. ipsilateral dysphagia, facial numbness, cranial nerve palsy e.g. Horner’s and contralateral limb sensory loss
How well did you know this?
1
Not at all
2
3
4
5
Perfectly
59
Q

Describe Dysarthria

A

Dysarthria is a disorder of speech due to disturbance of muscular control. Can be caused by UMN lesions of the cerebral hemisphere or LMN of the brain stem.

There may be some variation depending on the site of the lesion:

  • slurred and weak articulation with a weak voice is typical of pseudobulbar palsy from a stroke.
  • slurred, scanning and staccato speech caused by cerebellar lesions is typical of MS.
  • Dysrhythmic, dysphonic, monotonous voice can be due to disease of the extrapyramidal system in Parkinson’s.
How well did you know this?
1
Not at all
2
3
4
5
Perfectly
60
Q

Describe dysphasia

A

Dysphasia is impaired ability to understand or use language.it is due to a lesion of the dominant hemisphere and may include impaired ability to read, write and use gestures.

Wernicke’s dysphasia - lesions are located in temporal lobe specifically though to be in posterior section of superior temporal gyrus. Patients are unable to understand language speech is fluent but lacking content. Patients lack awareness of the speech difficulties and my get agitated

Broca’s dysphasia - lesions are located in the frontal lobe, also known as expressive dysphasia as patient are unable to speak fluently, difficult to understand but comprehension is good. patients are aware of their speech difficulties.

Conduction dysphasia - lesions around the arcuate fasciculus, posterior parietal and temporal regions. leads to naming deficits inability to repeat non meaningful words, although there is apparently normal speech comprehension and production.

Deep dysphasia - lesions are in the temporal lobe, symptoms are word repetition problems.

-Global dysphasia - occurs with extensive damage to basal ganglia, thalamus, symptoms are extensive with generalised deficits in comprehension, repetition, naming and speech production.

How well did you know this?
1
Not at all
2
3
4
5
Perfectly
61
Q

Name some of the visual fields defects and their causes

A

Bitemporal hemianopia - if upper quadrant affected more than lower = inferior chiasmal compression commonly due to pituitary tumour, if opposite way then superior chiasmal compression, commonly a cranipharyngioma.

Homonymous hemianopia with macular sparing = lesion of occipital cortex. Homonymous hemianopia with incongruous defects tends to be optic tract lesion where as congruous defects point towards optic radiation of occipital lesion.

Homonymous quadrantopias:
-superior = lesion of temporal lobe
-inferior = lesion of parietal lobe
Remember PITS Posterior-Inferior, Temporal-Superior

How well did you know this?
1
Not at all
2
3
4
5
Perfectly
62
Q

Describe the spinothalamic tracts and their journey through the CNS.

A

Spinothalamic tracts carries information related to pain, temperature, non-discriminative touch, and pressure.

Primary afferent fibres synapse in the contralateral dorsal horn, secondary neurones decussate through the ventral white commissure to the spinothalamic tract which lies ventral and lateral to the ventral horn.

From here the tract ascend, and in the brainstem fibres run in close proximity to the medial lemniscus and are known as the spinal lemniscus.

The majority of secondary fibres terminate in the ventral posterior nucleus of the thalamus, contacting third-order thalamocortical neurones that project to the somatosensory cortex.

How well did you know this?
1
Not at all
2
3
4
5
Perfectly
63
Q

Describe the dorsal columns and their journey through the CNS.

A

The dorsal columns are split into two tracts, the fasciculus gracilis (medial) and the fasciculus cuneatus (lateral). The tracts carry impulse concerned with proprioception and discriminative touch.

The dorsal columns contain axons of primary afferent neurones that have entered the cord through the dorsal roots. The fasciculus gracilis consits of fibres that join the cord at the sacral, lumbar and lower thoracic levels (lower limb) and the fasciculus cuneatus those from upper thoracic and cervical roots (upper limb). They travel on the ipsilateral side to the medulla oblongata where they terminate upon second-order neurones in the nucleus gracilis and cuneatus.

Second-order accons then decusste in the medulla as internal arcuate fibres and thereafter ascend through the brain stema s the medial lemniscus to terminate in the ventral posterior nucleus of the thalamus.

Third-order thalamocortical neurones in turn project to the somatosensory cortex located in the postcentral gyrus of the parietal lobe.

How well did you know this?
1
Not at all
2
3
4
5
Perfectly
64
Q

Describe the corticospinal tracts and their journey through the CNS.

A

A descending spinal tract concerned with the control of voluntary, discrete, skilled movements, especially those of the distal parts of the limbs

Primary neurones arise from motor and sensory parts of the cortex but the largest axons arise from betz cells in the premotor cortex in the precentral gyrus.

Corticospinal axons leave the cerebral hemisphers by passing through the corona radiate and internal capsule to entre the crus cerebri of the midbrain. Having passed through the the ventral portion of the pons, fibres reach the medulla oblongata where they form the pyrmiadal tracts on the ventral surface. In the caudal medulla majority of fibres decussate and travel down to cord in lateral corticospinal tracts however some remain ipsilateral as the ventral corticospinal tract which decussate lower down the cord.

How well did you know this?
1
Not at all
2
3
4
5
Perfectly
65
Q

Describe normal pressure hydrocephalus, its signs and symptoms, investigations, and treatment

A

Type of brain malfunction caused by decreased absorption of CSF typically presenting with the triad, ataxia, urinary incontinence and dementia. The dementia is predominantly frontal lobe in nature i.e. presents in the form of apathy, forgetfulness inertia, inattention.

Investigations:

  • CT or MRI to exclude SOLs, shows enlarged ventricles
  • lumbar puncture and evaluation of response to CSF removal.

Treatment:
-Surgical implantation of a ventriculoperitoneal shunt if clinically fit for surgery.

How well did you know this?
1
Not at all
2
3
4
5
Perfectly
66
Q

Describe stroke, its causes, risk factors, signs, and management

A

Strokes result from ischaemia infarction of bleeding into part of the brain, and manifest by rapid-onset of focal CNS signs and symptoms

Causes:

  • Small vessel occlusion, cerebral microangiopathy of thrombosis insitu
  • Cardiac emboli e.g. AF, endocarditis, MI
  • Atherothromboembolism e.g. from carotids
  • CNS bleeds e.g. hypertension, trauma, aneurysm rupture, anticoagulation, thrombolysis.
  • sudden BP drop
  • carotid artery dissection
  • vasculitis

Risk factors: hypertension, DM, heart disease, PVD, past TIA, carotid bruit, the pill, hyperlipidaemia, alcohol abuse, pro-thrombotic disorders

Signs: sudden onset with progression over hours

  • Cerebral infarcts: the following criteria should be assessed 3 of the below defines a TACI (middle and anterior cerebral arteries) and 2 of the below defines a PACI (smaller arteries of anterior circulation)
    1) Unilateral hemiparesis and/or hemisensory loss of the face, arm and leg.
    2) Homonymous hemianopia
    3) high cognitive dysfunction e.g. Dysphasia.
  • Brainstem infarcts: quadriplegia, locked-in syndrome
  • Lacunar infarcts: in basal ganglia, thalamus, and pons. ataxic hemiparesis, pure motor, pure sensory, sensorimotor and dysarthria/clumsy hand.

Management:

  • Protect airway to avoid hypoxia/aspiration
  • Pulse, BP, ECG - exclude AF, haemodynamically stable treat hypo not hypertension
  • BM
  • Urgent CT/MRI if thrombolysis considered, cerebellar stroke, unusual presentation,
  • Consider thrombolysis if haemorrhagic stroke excluded and onset of symptoms less than 4.5hrs ago
  • NBM keep hydrated assess speech and swallow
  • Antiplatelet agents once haemorrhagic stroke excluded, aspirin 300mg for 2 weeks mono therapy then after 2 weeks dual antiplatlets.
  • do not routinely anti-coagulate in ischaemic stroke due to risk of haemorrhagic transformation
  • refer to stroke unit asap
  • consider statin if hyperlipidaemia, controlling BP but not acutely unless thrombolysed in which case aim BP less than 180/110. In Haemorrhage aim BP less than 160/110. If no thrombolysis do not routinely treat BP unless over 220mmHg systolic. Seek Stroke nurse/specialist support.
How well did you know this?
1
Not at all
2
3
4
5
Perfectly
67
Q

Describe Guillain-Barre syndrome, its presentation, triggers, and management

A

Also known as acute inflammatory demyelinating polyneuropathy.

Presentation: Typically presents a few weeks after an infection with a symmetrical ascending muscle weakness. The trigger causes antibodies which attack nerves, though in 40% no cause is found. It may advance quickly, affecting all four limbs at once, and can lead to paralysis. There is a progressive phase of up to 4 weeks followed by recovery.

Management:

  • lumbar puncture, CSF shows increased protein in absence of raised WCC.
  • FVC monitoring 4 hourly with transfer to ITU if respiratory involvement.
  • Cardiac monitoring required for risk of bradycardia or SVT
  • Ventilate sooner rather than later e.g. if FVC less than 1.5L or signs of respiratory failure
  • IV immunoglobulin 0.4g/kg/24h for 5d

Triggers include campylobacter jejuni, CMV, mycoplasma, zoster, HIV, EBV, vaccinations.

How well did you know this?
1
Not at all
2
3
4
5
Perfectly
68
Q

What is apraxia

A

A motor disorder cause by damage to the posterior parietal cortex leading to difficulty with motor planning to perform tasks or movements when asked.

How well did you know this?
1
Not at all
2
3
4
5
Perfectly
69
Q

Describe Vaso-vagal syncope, typical history,

A

Due to reflex bradycardia +/- peripheral vasodilatation provoked by emotion, pain, fear or standing too long. Onset is over seconds (not instantaneous), and is often preceded by nausea, pallor, sweating and closing in of visual fields (pre-syncope). It cannot occur when lying down. LOC for around 2 mins. Brief clonic jerking of the limbs may occur but there is no stiffening or tonic/clonic sequence. Urinary incontinence is uncommon but may occur, and there is no tongue-biting. Recovery is rapid.

How well did you know this?
1
Not at all
2
3
4
5
Perfectly
70
Q

Describe a circumducting gait and its causes

A

Patient stands with unilateral weakness on the affected side, arm held in flexion, adducted and internally rotated, leg extended on same side with plantar flexion of foot and toes. When walking the patient will hold his or her arm to onside and drag their affected leg in a semicircle (circumduction) due to weakness of distal muscles and extensor hypertonia.

This is most commonly seen in stroke.

How well did you know this?
1
Not at all
2
3
4
5
Perfectly
71
Q

describe a high-stepping gait and its causes

A

Seen in patients with foot drop (weakness of foot dorsiflexion), the cause of this gait is due to an attempt to lift the leg high enough during walking so that the foot does not drag on the floor.

If unilateral, causes include peroneal nerve palsy and L5 radiculopathy. If bilateral, causes include amyotrophic lateral sclerosis, Charcot-Marie-Tooth disease and other peripheral neuropathies including diabetic neuropathy.

How well did you know this?
1
Not at all
2
3
4
5
Perfectly
72
Q

Describe a myopathic (waddling) gait and its causes.

A

Hip girdles muscles are responsible for keeping the pelvis level when walking. If you have weakness on one side, this will lead to a drop in the pelvis on the contralateral side of the pelvis while-walking (Trendelenburg sign). with bilateral weakness, you will have drooping of the pelvis on both sides during walking leading to waddling

This gait is seen in patients with myopathies such as muscular dystrophy.

How well did you know this?
1
Not at all
2
3
4
5
Perfectly
73
Q

Describe the role of the hippocampus

A

Located in the medial temporal lobe of the brain, and part of the limbic system, the hippocampus plays important role in short-term memory, especially the conversion of short-term memory to long term, and spatial navigation

How well did you know this?
1
Not at all
2
3
4
5
Perfectly
74
Q

What is allodynia and name some causes?

A

Refers to triggering of pain response from stimuli which do not normally provoke pain.

Causes: Neuropathies, complex regional pain syndrome, fibromyalgia, migraine, spinal cord injury.

How well did you know this?
1
Not at all
2
3
4
5
Perfectly
75
Q

Describe Migraine, it’s symptoms, treatment, and prevention

A

A common cause of recurrent acute headaches.

Symptoms: Diagnostic criteria requires at least 3 attacks, of which last 4-72hours duration, and have at least 2 of the following characteristics; Unilateral, Moderate pain, pulsating quality, avoidance of physical activity. They must also have at least one of the following symptoms N+V, photophobia, phonophobia. And finally must not be attributable to another disorder. Classically headaches are preceded by an aura lasting 15-20minutes with onset of headache within an hour.

Aura:
Precedes headache by minutes and may persist during. May be visual such as chaotic lines or dots, visual distortion or scotomata, or hemianopia. May also be somatosensory such as paraesthesiae spreading from fingers to face. Motor aura such as dysarthria, ataxia, opthamolplegia or hemiparesis.

Treatment:

  • NSAIDS (e.g. Ketoprofen 100mg PO, or dispersive aspirin 900mg/6h) are good as there is reduced chance of developing medication-misuse headache.
  • Triptans such as rizatriptan may be used. They are CI if IHD, coronary spans, uncontrolled hypertension, recent lithium, SSRIs, or ergot use. SEs include arrhytmias and angina+/- MI
  • Ergots may also be used such as ergotamine 1mg PO can cause gangrene and vascular damage. CI in those taking pill, PVD, IHD, pregnancy, breastfeeding, hemiplegic migraine, raynauds.
  • Botulinum toxin type A injection may be used in chronic migraine headaches on more than 14days a month if no response to 3prophylactic drugs.

Prevention:

  • Remove triggers I.e. CHOCOLATE, Hangovers, Orgasms, Cheese, Oral contraceptives, Lie-ins, Alcohol, Tumult, Exercise.
  • If frequent I.e. More than 2 a month 1st line is propanolol 40-120mg/12h, amitriptyline 10-75mg nocte, or topiramate 25-50mg/12h
  • 2nd line = valproate, pizotifen, gabapentin, pregabalin,
How well did you know this?
1
Not at all
2
3
4
5
Perfectly
76
Q

Describe Meralgia Paraesthetica, its symptoms, and management.

A

An entrapment syndrome of the lateral femoral nerve.

Symptoms: causes burning or numbness down the upper lateral aspect of the thigh. Pain can be reproduced by deep palpation just below the upper lateral aspect of the thigh.

Management:

  • weight loss may cure the condition in the case of obesity
  • otherwise NSAIDs may be trialled
How well did you know this?
1
Not at all
2
3
4
5
Perfectly
77
Q

What is cushings triad?

A

It is a sign of increased Intracranial pressure, it is a triad of Hypertension, bradycardia and irregular breathing

How well did you know this?
1
Not at all
2
3
4
5
Perfectly
78
Q

Describe Cerebral Oedema, is warning signs, and management.

A

It is a big threat and is a complication of childhood DKA. Usually presents as sudden CNS deterioration after an initial improvement.

Warning signs: headache, cushings triad (Bradycardia, hypertension, irregular breathing), restlessness, irritability, focal neurology such as CN palsies, papilloedema.

Management:
-Call your senior, exclude hypoglycaemia, mannitol 0.25-1.5g/Kg IVI, retrial IV maintenance fluids by 1/2 and replace deficit over 72 hours. Move to PICU and do CT.

79
Q

Describe Tuberous Sclerosis and some of its features.

A

Also known as bourneville’s disease, it is a genetic condition of autosomal dominant inheritance. Like Neurofibromatosis the majority of features seen are neuro-cutaneous.

Cutaneous features:

  • depigmented ‘ash-leaf’ spots which fluoresce under UV light
  • roughed patches of skin over lumbar spine (shagreen patches)
  • adenoma sebaceum (angiofibromas), butterfly distribution over nose
  • subungual fibromyalgia
  • cafe-au-lait spots may be seen

Neurological features:

  • developmental delay
  • epilepsy, infantile spasms
  • intellectual impairment.

Other features include, retinal haramtomas, rhabdomyomas of heart, polycystic kidney disease, renal angiomyolipomata, polyps, peutz-Jeghers, lymphagniomyomatotsis

80
Q

Describe Cerebral Palsy, it’s presentation, causes, classification, and management

A

A chronic disorder of posture and movement caused by non-progressive CNS lesions sustained before 2yrs old, resulting in delayed motor development and evolving CNS signs, learning disability and epilepsy.

Presentation: Abnormal tone in early infancy, delayed motor milestones, abnormal gait, feeding difficulties. Associated non-motor problems include learning difficulties, epilepsy, squinted and hearing impairment.

Causes: Antenatal e.g. Cerebral malformation and congenital infection, intrapartum e.g. birth asphyxia/trauma, postnatal e.g. IVH, meningitis, trauma, Kernicterus.

Classification:

  • spastic: hemiplegic, diplegic, quadriplegic
  • dyskinetic
  • ataxic
  • mixed

Management:

  • MDT approach
  • treatments for spasticity include oral diazepam, intrathecal baclofen
  • anti-epileptics as required.
81
Q

Describe Multiple System Atrophy and its symptoms.

A

A rare neurodegenerative disorder caused by cell loss in certain areas of the brain and spinal cord. Characterised by widespread Glial cytoplasmic inclusions. Typically occurs between 50-70yrs.

Symptoms: Parkinsonism, autonomic disturbance (atonic bladder, postural hypotension, constipation), cerebellar signs

82
Q

Describe Duchenne’s Muscular Dystrophy, its symptoms, investigations and management.

A

Commonest of the muscular dystrophies, it is a genetic disease with progressive degeneration and weakness of specific muscle groups. It is an X-linked recessive mutation in the dystrophin gene. Patients tend to die due to cardiomyopathy and respiratory failure. Can affect females 10-20% developing full blown phenotype with some having cardiomyopathy only.

Symptoms: Presents around 4yrs with clumsy walking and then difficulty in standing, and respiratory failure. Pseudo hypertrophy is seen especially in calves.

Investigations: CK is raised 40-fold.

Management:

  • no specific treatment
  • physio and occupational input
  • ventilation may be needed.
83
Q

Describe Multiple Sclerosis, the types, its symptoms, and management.

A

Chronic autoimmune demyelinating disorder. 3 times more common in women, most commonly diagnosed between 20-40.

Types:

  • relapsing-remitting disease (most common 85%) acute attack (last 1-2mnths) followed by periods of remission.
  • secondary progressive disease, relapsing remitting patients who have deteriorated and have developed neurological signs between relapses. Gait and bladder disorders are generally seen.
  • primary progessive, deterioration from onset, more common in older people.

Symptoms: usually monosymptomatic: Lethargy, unilateral optic neuritis (pain on eye movement and rapid loss of central vision), optic atrophy, ophthalmoplegia, pins/needles, numbness, Trigeminal neuralgia, spastic weakness, cerebellar ataxia, tremor, urinary incontinence, sexual dysfunction, intellectual deterioration.

Management:

  • vitamin D status related to prevention of MS and fewer symptoms and lesions ensure levels above 50nmol/L
  • Acute relapse, high dose steroids may be given for 5 days to shorten the length of relapse (PO methylprednisolone has been shown to be non-inferior to IV methylprednisolone)
  • Beta-interferon reduce relapse rate by up to 30%, also dimethyl fumarate
  • other drugs include, galatiramir acetate, natalizumab, fingolimod
84
Q

Describe Myotonic Dystrophy, its types, and features.

A

AKA dystrophia myotonica, an inherited autosomal dominant, myopathy with features developing at around 20-30 years old. It affects skeletal, cardiac and smooth muscles.

Types:
There are two main types DM1 and DM2.

  • DM1 present with more distal weakness caused by trinucleotide CTG repeat at the end of DMPK gene on chromosome 19
  • DM2 is more proximal, caused by a tetranucleotide repeat expansion of the (CNBP) ZNF9 gene on chromosome 3.

Symptoms:
-DM1 (Distal weakness, Autosomal Dominant, Diabetes, Dysarthria)

General Features: Myotonic fancies (long, haggard appearance), frontal balding, bilateral ptosis, cataracts, dysarthria, myotonia (tonic spasm of muscle), weakness of arms and legs, mild mental impairment, diabetes Mellitus, testicular atrophy, heart block, cardiomyopathy, dysphagia.

85
Q

Describe Trigeminal Neuralgia, its symptoms, and management.

A

Symptoms: Paroxysms of intense, stabbing pain, lasting seconds, in the Trigeminal nerve distribution. It is unilateral, typically affecting mandibular or maxillary divisions. The face screws up with pain. May be triggered by washing affected area, shaving, eating, talking, dental prostheses.

Management:

  • MRI is required to exclude secondary causes e.g. Aneurysm, tumour, MS
  • Carbamazepine (100mg 1-2times a day), lamotrigine, phenytoin or gabapentin may be tried.
86
Q

Describe Syringomyelia, its symptoms, and management.

A

A condition in which there is a fluid-filled tubular cyst (syrinx) within the central usually cervical spinal cord. It can elongate and expand to affect the Brainstem (syringobulbia).

Symptoms: Dissociated sensory loss ( absent pain and temperature, with preserved light touch, vibration and joint position sense) in a root distribution. Horner’s syndrome, UMN leg signs. Body asymmetry, If extension into Brainstem nystagmus, tongue atrophy, dysphagia, palatal weakness, Vth sensory loss.

Management:

  • MRI imaging
  • Neurosurgical review.
87
Q

Describe insominia, its causes, and management

A

Causes:

  • self limiting e.g. Jet lag, stress, shift work, old age
  • psychological e.g. Depression, anxiety, mania, grief, agitation/psychosis
  • Organic e.g. Drugs (caffeine, nicotine withdrawal), nocturia, pain, itch, tinnitus, sleep apnoea, restless leg syndrome (check ferritin).

Management:

  • advise re sleep hygiene, no daytime naps, into turn in until feel sleepy, regular bedtime routines, keep a room for sleep do not work in it, less caffeine.
  • Hypnotic drugs for a few nights only, e.g. Zopiclone warn risk of addiction and tolerance. And rebound insomnia on withdrawal, no heavy machinery use or driving.
88
Q

What movements are the different myotomes responsible for?

A

Shoulder: Abduction = C5, Adduction = C5-C7
Elbow: Flexion = C5-C6, Extension = C7
Wrist: Flexion = C7-C8, Extension = C7
Fingers: Flexion = C8, Extension = C7, Abduction = T1
Hip: Flexion = L1-L2, Adduction = L2-L3, Extension = L5-S1
Knee: Flexion = L5-S1, Extension = L3-L4
Ankle: Dorsiflexion = L4, Eversion = L5-S1, Plantarflexion = S1-S2
Toe: Big toe extension = L5

89
Q

Describe Myasthenia Gravis, its features, investigations, and management.

A

An autoimmune disease mediated by antibodies to nicotinic acetylcholine receptors. More common in women under 50 or men over 50, associated with thyimic tumour/atrophy, RA and SLE.

Features: Increasing muscular fatigue, typically affecting (in order) ocular, bulbar, face, neck, limb girdle, trunk. Look for ptosis, Diplopia, myasthenic snarl on smiling, ‘peek sign’ (fatigue on trying to keep eyes closed). Reflexes are normal.

Investigations: Anti-AChR (90%) if negative look for Anti-MuSK (Muscle specific tyrosine kinase). CT of thymus. Neurophysiology if unsure.

Management:

  • Weakness is made worse by pregnancy, hypokalaemia, infection, exercise, gentamicin, opiates, tetracycline, quinine, procainamide, beta-blockers.
  • Symptom control with anticholinesterase e.g. Pyridostigmine (SEs increased salivation, lacrimation, sweats, vomiting, miosis
  • Immunosuppression (Prednisolone, Azathioprine, cyiclosporin, mycophenolate) treat relapses with prednisolone, give osteoporosis prophylaxis, For treatment resistant Tacrolimus, cyyclophoshamide or rituximab, or periodic IVIg
  • in myasthenia crisis, monitor FVC, treat with plasmapheresis or IV Ig
90
Q

Describe Lambert-Eaton Myasthenic Syndrome, its features, and management.

A

Can be a paraneoplastic syndrome (from small cell lung cancer) or autoimmune. Caused by an antibody to pre-synaptic membranes voltage gated Ca-channels (Anti P/Q type VGCC)

Features: Similar to MG but there is gait difficulty before eye signs, autonomy involvement (dry mouth, constipation, impotence), hyporeflexia and weakness, which improve after exercise.

Management:

  • 3,4-diaminopyridine
  • IV Ig
  • regular CXR/ HRCT as symptoms may precede cancer by over 4 years.
91
Q

What are the causes of mixed UMN and LMN lesions

A

Fred’s Tabby Cat Seeks Mice

Friedrich's Ataxia
Tabes dorsalis (syphilis)
Cervical spondylitis
Subacute degeneration of the cord
Motor neurone disease
92
Q

Describe Bulbar Palsy, its signs, and causes.

A

Bulbar palsy is a LMN lesion of cranial nerve IX, X and XII

Signs: flaccid, fasiculating tongue, jaw jerk is normal or absent, speech is quiet, hoarse, or nasal, gag reflex is absent.

Causes: MND, Syringobulbia, Guillian-Barre, poliomyelitis, central poninte myelinolysis, brainstem tumours, myasthenia gravis

93
Q

Describe pseudobulbar palsy, its signs and causes.

A

Pseudobulbar palsy is an UMN lesion of cranial nerves IX, X and XII.

Signs: slow tongue movements, increased jaw jerk, increased gag reflex, Donald Duck speech, pseudobulbar affect causing weeping unprovoked or sudden giggling.

Causes:

  • bilateral CVAs affecting internal capsule
  • MS
  • MND
  • High brainstorm tumours
  • Head injury
94
Q

What are the signs of an anterior communicating artery aneurysm?

A

Most common aneurysm of the circle of Willis can cause visual field defects such as bitemporal hemianopia due to compression of the optic chiasm, psyhycopathology and frontal lobe pathology.

95
Q

Describe Charcot-Marie-Tooth disease, its features, and management.

A

Hereditary motor and sensory neuropathy, most commonly inherited neurological disorder affecting approx 1/2500.

Features: Foot drop, hammer toe, wasting of muscle of lower prt of legs (inverted champagne bottle), weakness in hands and forearms, loss of touch sensation in distal limbs. Pes cavus and pes planus are also seen.

Management:

  • incurable
  • ankle-foot orthoses to help with foot drop
96
Q

Describe Mono-neuritis multiplex and its causes

A

A term used for neuropathies if two or more peripheral nerves are affected.

Causes: WARDS PLC
Wegner's granulomatosis (Granulomatosis with Polyangitis)
Amyloidosis/ AIDS
Rheumatoid Arthritis
Diabetes Mellius
Sarcoidosis
Polyarteritis Nodosa
Leprosy
Carcinomatosis
97
Q

What are the causes of a motor peripheral neuropathy?

A

Guillian Barre,
Lead poisoning
Charcot-Marie-tooth
Chronic Inflammatory Demyelinating Polyneuropathy

98
Q

Describe Huntington’s disease, its features and management.

A

Autosomal dominant, progressive neurodegenerative disorder presenting in middle age. Due to expansion of CAG repeat on chromosome 4. 10% are new mutations.

Features: Often prodroml pahse of mild symptoms (irritability, depression, incoordination), progresses to chorea (random, jerky uncontrollable movements), dementia, seizures and death.

Management:
Dopamine antagonists to control chorea
No treatment prevents progression
Counselling for families and patient, anticipation (apparent at earlier age with successive generations)

99
Q

Describe FascioScapuloHumeral muscular Dystrophy and its features.

A

Almost as common as duchenne’s. Autosomal dominant inheritance onset is around 12-14yrs. Around 20% require a wheelchair by 40yrs.

Features: inability to puff out cheeks, difficulty raising arms above head, weakness of face, shoulder and upper arms, bilateral foot drop, scapular winging

100
Q

What are the causes of a sensorimotor peripheral neuropathy?

A
Alcohol
B12/folate deficiency
CKD
Diabetes
Everything else e.g. Vasculitis, HIV, trace elements (e.g. zinc, copper, chromium deficiencies)
101
Q

Describe Intranuclear opthalmoplegia, its features and causes.

A

Causes by a lesion in the medial longitudinal fasciculus.

Features: When patients gaze is directed away from the side of the lesion, the ipsilateral (adducting) eye will not adduct and the contralateral (abducting) eye demonstrates horizontal nystagmus. Patients usually do not complain of diplopia, and when the eyes are tested independently medial rectus function is shown to still be present.

Causes:

  • MS (likely cause in adulthood/middle age often bilateral)
  • Vascular brainstem lesion (likely in elderly or with vascular RFs)
  • Pontine glioma (likely cause in children)
  • Inflammatory encephalitis
102
Q

What are some causes of bilateral facial nerve palsy?

A

Sarcoidosis
Vasculitis
Lyme disease
Guillian-Barre

103
Q

Describe Miller-fisher syndrome and its features

A

A variant of Guilliam barre that present in a descending fashion, associated with Anti-GQ1B.

Features: classical triad of opthalmoplegia, ataxia, areflexia

104
Q

Describe Lumbar Canal Stenosis and its features.

A

Compression of the lumbar spinal cord due to stenosis.

Features: Low back pain and leg weakness that is relived when bending forward, walking uphill or stairs, or cycling.

105
Q

Describe Weber’s Syndrome and its features

A

Midbrain stroke syndrome, (Brainstem strokes have crossed finding with motor or sensory finding on the contralateral side of the body and cranial nerve findings ipsilateral) caused by occlusion of the paramedian branches of the basilar artery

Features: Ipsilateral surgical IIIrd nerve palsy, and contralateral hemiplegia or hemiparesis

106
Q

Describe Von Hippel-Lindau syndrome and its features.

A

Autosominal dominant condition due to mutations in the VHL gene on chromosome 3. Results in the development of haemangioblastomas, which are CNS tumours that originate form the vascular system. Can also develop renal cell carcinoma and cysts in the liver, kidney and pancreas.

Features: Cerebellar syndrome, Ectopic EPO production (polycythaemia), phaeochromocytoma (flushing, headache, palpitations)

107
Q

Describe Motor Neuron Disease, the types, features, and managment.

A

Cluster of major degenerative disease characterised by selective loss of neurons in the motor cortex, cranial nerve nuclei and anterior horn cells. UMN + LMN with no sensory loss never affecting eye movements.

Types + Features:

  • Amyotrophic lateral sclerosis: affects motor cortex and anterior horn cells. weakness, UMN + LMN signs worse prognosis if bulbar onset, increased age, decreased FVC
  • progressive bulbar palsy: only affect cranial nerves IX-XII
  • progressive muscular atrophy: anterior horn cell only thus no UMN signs, affects distal muscle groups before proximal.
  • Primary lateral sclerosis: los of betz cells in motor cortex, thus mainly UMN signs, spastic leg weakness and pseudobulbar palsy

Management:

  • MDT approach, neurologist, palliative nurse, hospice, physio, speech therapist, OT, dietician, social services.
  • Treat symptomatically
  • Riluzole prevents stimulation of glutamate receptors used mainly in amyotrophic lateral sclerosis and prolongs life by around 3 months.
108
Q

Describe Post-lumbar puncture headache, its features, factors which contribute to headache, and management.

A

Features:

  • usually develops within 24-48hr following LP but may occur up to one week later
  • may last several days
  • worsens with upright position
  • improves with recumbent position

Aggravating Factors:

  • Increased needle size
  • Direction of bevel
  • not replacing the stylet
  • Increased number of LP attempts

Management:

  • Supportive, rest and analgesia
  • if pain continues more than 72 hours then treatment is needed to prevent subdural haematoma, treatment options include, blood patch (A surgical procedure that uses autologous blood in order to close one or many holes in the dura mater), epidural saline, and intravenous caffeine.
109
Q

Describe Transient Global Amnesia

A

Present widths Transient loss of memory function

Patients may appear anxious and repeatedly ask the same question

Patients have no recall of events after the attack

Aetiology is unknown thought to be due to Transient ischaemia to the thalamus in particular the amygdala and hippocampus.

110
Q

What are the neurological features of a Parietal Lobe lesion?

A

Parietal Lesions depend on whether the lesion is dominant (Left), Non-dominant (Right) or bilteral:

Unilateral lesions: (right or left) may cause:

  • Contralateral hemisensory loss and sensory inattention
  • Inferior homonymous quadrantopia
  • Abolotion of optokineticnystagmus.

Unilateral dominant (usually left) may cause:

  • Apraxia (Difficulty with motor planning to perform tasks)
  • Astereognosis aka Tactile agnosia (the inability to identify an object by active touch of the hands)
  • Gerstmann’s Syndrome (Lesion of dominant parietal): alexia (Difficulity reading) , acalculia (difficulty performing calculations), finger agnosia (difficulty recognising the fingers), right-left disorientation.

Unilateral non-dominant (usually right) may cause:

  • Predominantly visuospatial dysfunction
  • Constructional apraxia (Ability to copy a drawing)
  • Anosognosia and dressing apraxia.
111
Q

What are the neurological features of a Occipital lobe lesion?

A

Homonymous hemianopia (with macula sparing)
Cortical Blindness
Visual Agnosia

112
Q

What are the neurological features of a Temporal Lobe lesion?

A

Wernicke’s Aphasia (Receptive aphasia)
Superior Homonymous Quadrantopia
Auditory Agnosia
Prosopagnosia (difficulty in recognising faces)

113
Q

What are the neurological features of a Frontal Lobe lesion?

A
Broca's Aphasia (Expressive aphasia)
Disinhibition
Perseveration
Anosmia
Inability to generate a list
114
Q

What are the neurological features of a cerebellum lesion?

A

Midline lesions causes gait and truncal ataxia

Hemisphere lesions causes intention tremor, past-pointing, dysdiadokokinesis and nystagmus

115
Q

Describe Korsakoff’s syndrome and its features.

A

Marked memory disorder often seen in alcoholics, thiamine deficiency causes damage and haemorrhage to the mammillary bodies of the hypothalamus and the medial thalamus. It often follows on from untreated Wernicke’s encephalopathy

Features:

  • anterograde amnesia (inability to acquire new memories)
  • Retrograde amnesia
  • confabulation
116
Q

Describe Reye’s syndrome and its features.

A

Reye’s syndrome is a severe progressive encephalopathy affecting children that is accompanied by fatty infiltration of the liver, kidneys and pancreas. There is a known association with aspirin use and a viral cause has been postulated. Peak incidence is 2 years

Features:

  • May be history of preceding viral illness
  • Encephalopathy: confusion, seizures, cerebral oedema, coma
  • Fatty infiltration of liver, kidneys and pancreas
  • hypoglycaemia
117
Q

Describe Friedrich’s ataxia and its features.

A

Most common of the early-onset hereditary ataxias. Autosomal Recessive, trinucleotide repeat disorder characterised by a GAA repeat in the X25 gene on chromosome 9. Unusual in trinucleotide repeat disorders in not demonstrating anticipation.

Typical age of onset is 10-15yrs. Gait ataxia and kyphoscoliosis are most common presenting features:

Neurological features:

  • Absent ankle jerks/extensor plantars
  • Cerebellar ataxia
  • Optic atrophy
  • Spinocerebellar tract degeneration - Decreased vibration and proprioception, pyramidal weakness

Other features:

  • HOCM (90%, most common cause of death)
  • Diabetes Mellitus (10-20%)
  • High-arched palate
  • deafness
118
Q

Describe Cervicial Spondylosis, its features, and management.

A

Aka Degenerative cervical myelopathy.

Features:

  • Pain (affecting the neck or upper or lower limbs)
  • Loss of motor function (loss of digital dexterity, arm of leg weakness/stiffness)
  • loss of sensory function causing numbness
  • loss of autonomic function (urinary or faecal incotinence or impotence)

Management:

  • MRI of cervical spine is gold standard test.
  • Urgent Referral for to neurosurgery or orthopaedic spinal surgeons
  • decompressive surgery is only effective treatment.
119
Q

What are Dawson Fingers?

A

A radio graphic feature depicting demyelinating plaques through the corpus callosum arranged at right angles along medullary veins (Callososepal location). They are a relatively specific sign for Multiple Sclerosis which presents at T2 Hyperintensities.

120
Q

Describe Kluver-Bucy syndrome and its features.

A

A rare neurological condition caused by a brain lesion in the Amygdala.

Features: Hypersexuality, Hyperorality, hyperphagia, visual agnosia.

121
Q

Describe Osmotic demyelination syndrome and its features.

A

Aka Central pontine myelinolysis, a demyelination Syndrome caused by the rapid correction of chronic hyponatraemia.

Features: Quadriparesis and bulbar palsy, seizures, confusion, movement disorders.

Diagnosis with MRI Brain which can show Tripod sign.

122
Q

Describe Intracranial Hypotension, its features and causes.

A

Aka Craniospinal hypotension, defined as CSF pressure less than 7cmH20.

Features:
Postural headache better when lying flat, nausea + vomiting, neck pain, visual and hearing disturbances, vertigo.

Causes:

  • Primary - Spontaneous intracranial hypotension
  • secondary: iatrogenic (lumbar puncture or surgery), over shunt due to diversion devices, traumatic.
123
Q

Describe Marchiafava-bignami disease and its features.

A

Mostly seen in chronic alcoholics it is attributed to deficiency’s in all eight types of vitamin B leading to neurosis and demyelination of the corpus callosum

Features:
Non specific e.g. motor or cognitive disturbances, hemispheric disconnection syndrome or seizures. Radiologically is classically involves the central layers of the corpus callosum with relative sparing of the dorsal and ventral extremes which may be seen as a sandwhich sign on Sagittal MRI.

124
Q

Describe Copper deficiency and its causes and its features

A

A rare haematological and neurological disorders. May manifest in parallel with vitamin B12 and other nutritional deficiencies. Copper is involved in normalised function of many enzymes such as cytochrome C oxidase a complex in mitochondrial electron transport chain (catalyses reactions for oxidative phosphorylation), Ceruloplasmin (catalyses reactions for iron transportation), Cu/Zn Superoxide dismutase (catalyses reactions for antioxidant and free radical scavenging and neutralisation) and amine oxidises (catalyses neurotransmitter synthesis)

Causes:

  • most common cause is malabsorption e.g. gastric bypass surgery
  • Zinc toxicity (zinc found in denture cream)
  • Menkes Disease genetic disorder of copper deficiency

Features:

  • Haematolgical manifestations e.g. myelodysplasia, anaemia, leukopenia, neutropenia.
  • Neurological manifestations e.g. sensory ataxia, spasticity, muscle weakness, optic neuropathy, myelopathy.
  • Bone marrow aspirate shows dysplasia, ring sideroblasts (Features shared with myelodysplastic syndrome) uniquely shows cytoplasmic vacuoles within red and white cell precursors.
125
Q

Describe Balint’s Syndrome and its features

A

An uncommon triad of severe neuropsychological impairments:

  • Simultanagnosia (Inability to perceive the visual fields as a whole)
  • Oculomotor ataxia (difficulty fixating the eyes)
  • Optic ataxia (inability to move the hand to a specific object by using vision)
126
Q

Describe Opsoclonus Myoclonus Syndrome, its features

A

Rare neurological disorder, thought to be autoimmune process. Affects 2-3% of children with neuroblastoma (accounts for 50% of cases) and has been reported to occur with coeliac disease. May be associated with Anti-Ri.
Features:
-Opsoclonus (Rapid, involuntary, multivectorial, unpredictable conjugate fast eye movements without intersaccadic intervals.
-Myoclonus (brief, involuntary twitching of a muscle or a group of muscles)
-cerebellar ataxia, both truncal and appendicular
-Aphasia (impairment of speech and comprehension of speech)
-Mutism
-Lethargy

127
Q

Describe Isaac’s Syndrome (Neuromyotonia), its causes and features.

A

A form of peripheral nerves hyperexcitability that causes spontaneous muscular activity resulting from resistive motor unit action potentials of peripheral origin. Symptoms are most prominent in the Calves, legs, trunk, and soft ice the face and neck.

Causes:

  • Acquired (Most common), typically caused by antibodies that bind to potassium channels on the motor nerve.
  • Paraneoplastic
  • Hereditary

Features:
-Muscle cramps, stiffness, myotonia-like symptoms (slow relaxation), associated walking difficulties, hyperhidrosis (excessive sweating), myokymia (quivering of a Muscle), fasciculations, fatigue, exercise intolerance, Myoclonus jerks.

128
Q

What is Hoffmann’s reflex?

A

Elected by a reflex test which can help verify the presence of absence of issues arising from the corticospinal tract. Also known as the finger flexor reflex.

The test involves loosely trapping the middle finger at the distal inter-phalangeal joint and then scratch down the terminal phalanx nail plate allowing the phalanx to flick up naturally. A positive response is seen when there is flexion of the terminal phalanx of the thumb on the same appendage.

Sign of UMN lesion (can be normal) e.g. MS, Stroke, Cervical myelopathy, cord compression

129
Q

Describe Herpes Simplex encephalitis, its features, investigations, and management.

A

HSV encephalitis characteristically affects the temporal an inferior frontal lobes. HSV-1 is responsible for 95% of cases.

Features: Fever, headache, psychiatric symptoms, seizures, vomiting, focal features e.g. aphasia, peripheral lesions have no relation to presence of HSV encephalitis.

Investigations:

  • CSF: Lymphocytosis, elevated protein
  • PCR for HSV
  • CT: Medial temporal and inferior frontal changes (e.g. petechial haemorrhages) normal in 1/3rd of patients.
  • MRI is betters
  • EEG Pattern: Lateralised periodic discharges at 2 Hz

Treatment: Intravenous aciclovir

130
Q

Describe Creutzfeldt-Jakob disease, its features, investigation.

A

CJD is a rapidly progressive neurological condition caused by prion proteins. These proteins induce the formation of amyloid folds resulting in tightly packed beta-pleated sheets resistant to pro teases.

Features: Rapid onset dementia, myoclonus.

Types:

  • Sporadic CJD accounts for 85% of cases, 10-15% are familial, mean age of onset is 65yrs.
  • New Variant CJD occurs in younger patients (average onset 25yrs) psychological symptoms such as anxiety, withdrawal, and dysphonia are most common presenting features. The prion protein is encoded on chromosome 20. Methionine homozygosity at codon 129 of the prion protein is a risk factor for developing CJD.

Investigations:

  • CSF is usually normal
  • EEG shows biphasic, high amplitude sharp waves (only in sporadic CJD)
  • MRI shows hyper intense signals in the basal ganglia (Hockey stick sign) and thalamus (Pulvinar sign).
  • Real-time Quaking Induced Conversion (RTQuIC) is a new sensitive and specific test for detecting protein 14.3.3 in sporadic CJD
131
Q

Describe Chorea and its causes

A

Chorea describe involuntary rapid jerky movements which often move from one part of the body to another. Slower, sinuous movement of the limbs is termed athetosis. Chorea is caused by damage to the basal ganglia, especially the caudate nucleus.

Causes:
-Hungtindon’s disease, Wilson’s disease, Ataxic telangiectasia
-SLE, anti-phospholipid syndrome
-Rheumatic fever: Sydenham’s chorea
-Drugs e.g. oral contraceptive pill, L-dopa, antipsychotics
-Neuroacanthocytosis
-Pregnancy: Chorea gravidarum
-Thyrotoxicosis
Polycythaemia Rubra Vera
-Carbon monoxide poisoning
-cerebrovascular disease
132
Q

What is Parkinsonian Malignant Syndrome?

A

A life-threatening condition that occasionally occurs in patients with abrupt cessation of anti-parkinsonian medications. It is a cause of neuroleptic malignant syndrome

133
Q

Describe the Reticular Activating System, its structure, and function

A

The ascending reticular activating system is a set of connected nuclei in the brains of vertebrates that is responsible for regulating wakefulness and sleep-wake transitions.

It is composed of several neuronal circuits connecting the dorsal part of the posterior midbrain and anterior pons to the cerebral cortex via distinct pathways through the thalamus and hypothalamus.

134
Q

Describe Froin’s Syndrome and its causes.

A

Co-existance of xanthocrhomia, high proteins level, and marked coagulation of CSF. Stagnation of the CSF within the thecae sac facilities exudation from the tumour itself and activation of coagulation factors.

Causes:

  • meningeal irritation e.g. Spinal meningitis
  • CSF flow blockage e.g. tumour mass or abscess
135
Q

Describe Acute Disseminated Encephalomyelitis

A

ADEM is an autoimmune Demyelinating disease of the central nervous system. It may also be termed post-infectious encephalomyelitis. Common infections include measles, mumps, rubella, varicella and small pox.

After a lag time of between a few days to 2 months there is an acute onset of multifocal neurological symptoms with rapid deterioration. Non specific signs such as headache, fever, nausea and vomiting may also accompany the onset of illness. Motor and sensory deficits are frequent and there may also be brainstem involvement including occulomotor defects.

There are no specific bio markers for the diagnosis of ADEM. MRI imaging may show areas of supra and infra-tectorial demyelination. Management involves IV Glucocorticoids and the consideration of IVIG

136
Q

Describe Progressive Supranuclear Palsy and its features

A

AKA Steele-Richardson-Olszewski syndrome. A ‘Parkinson-plus’ syndrome.

Features:

  • Impairment of vertical gaze (down gaze worse than up gaze - patients may complain of difficulty reading or descending stairs)
  • Parkinsonism
  • Falls
  • Slurring of speech, pseudobulbar palsy
  • Cognitive impairment - subcortical dementia
  • Axial rigidity
  • Signe d’applause - ask patient to clap hands three times they proceed to clap more than 3
137
Q

Describe Limbic (Autoimmune) Encephalitis, it types, features,

A

A form of encephalitis caused by autoimmunity it may be paraneoplastic or non-paraneoplastic (e.g. secondary to infection, autoimmune disorder, idiopathic). It is often classified according to the auto-antibody which causes the disease:

  • Anti-Hu (associated with small-cell lung carcinoma)
  • Anti-Ma2 (Associated with germ-cell tumours of the testis)
  • Anti-NMDAR (associated with tumour of the ovaries, commonly teratomas)
  • Anti-VGKC(LGI1, CASPR2, Contactin-2) (Not associated with tumours)

Features:

  • Subacute short-term memory deficit
  • Headache, irritability, sleep disturbance, delusions, seizures (Fascio-brachial dystonic seziures (characteristic of LGI1 autoantibodies))

Investigations:

  • CSF Analysis - Elevated Protein, Normal Glucose, Elevated Lymphocytes (But usually less than 100).
  • Auto-antibodies
  • MRI Brain - Increased T2 signal in one or both temporal lobes.

Management:

  • IVIG
  • Plasmapheresis
  • Corticosteroids, cyclophosphamide, rituximab.
138
Q

What are the different Prion diseases?

A

Human Prion Diseases:

  • Creutzfeldt-Jakob disease
  • Variant Creutzfeldt-jakob disease
  • Gerstmann-straussler-Scheinker Syndrome
  • Fatal Familial Insomnia
  • Kuru

Animal Prion Diseases:

  • Bovine spongiform Encephalopathy
  • Chronic Wasting Disease
  • Scrapie
  • Tranmissible Mink Encephalopathy
  • Feline spongiform encephalopathy
  • Ungulate spongiform encephalopathy
139
Q

Describe Refsum disease and its features

A

aka Phytanic acid deficiency, an autosomal recessive neurological disease that results in the over accumulation of phytanic acid in cells and tissues.

Features: cerebellar ataxia, peripheral neuropathy, Retinitis Pigmentosa, sensorineural hearing loss, ichthyosis

140
Q

Describe Hiriyama disease and its features

A

aka Monomyelic Amyotrophy or Non-progressive Juvenile Spinal Muscular Atrophy. An untreatable focal motor neurone disease that primarily affects young males in India and Japan. A type of cervical myelopathy

Features: Insidious onset muscular atrophy usually of the forearms and hands, which stabilises at a plateau after 2-5 years with no progression. There is no pain or sensory loss. Sparing of the brachioradialis

141
Q

Describe Kennedy’s Diseaseand its features

A

Aka Spinal and Bulbar Muscular Atrophy, a progressive neurodegenerative disease resulting in muscles cramps and progressive weakness due to degeneration of motor neurone in the brainstem and spinal cord. Associated with Mutation of Androgen Receptor Gene and is inherited in X-linked recessive fashion, caused by a CAG trinucleotide repeat in the first exon of the gene.

Features: (Lower motor neurones signs) Muscle cramps, progressive weakness. Early signs include weakness of the tongue and mouth muscles fasciculations and gradually increasing weakness of the limb muscles with wasting.

142
Q

Describe Trigeminal Autonomic Cephalgia

A

A group of primary headache disorders characterised by unilateral trigeminal distribution pain that occurs in association with prominent ipsilateral cranial autonomic features.

The group comprises of:

  • Cluster Headache
  • Paroxysmal Hemicrania
  • Hemicrania Continua
  • Short-lasting Unilateral Neuralgiform headache attacks with Conjunctival injection and Tearing (SUNCT syndrome).
143
Q

What are the different pathologies and causes of peripheral neuropathy?

A

Demyelinating pathology:

  • GBS
  • CIDP
  • Amiodarone
  • Hereditary sensorimotor neuropathies (HSMN) Type I
  • Paraprotein neuropathy

Axonal Pathology:

  • Alcohol
  • Diabetes Mellitus (May also be demyelinating)
  • Vasculitis
  • B12 Deficiency (may also be demyelinating)
  • Hereditary sensorimotor neuropathies (HSMN) type II
144
Q

Describe Neuromyelitis Optica, its features, investigations

A

aka Devic’s disease is monophonic or releasing-remitting demyelinating CNS disorder. Although previously thought to be a variant of multiple sclerosis it is now recognised to be a distinct disease, particularly prevalent in asian populations. It typically involves the optic nerves and cervical spine, with imaging of the brain frequently normal (A distinguishing feature between MS). Vomiting is also the most common presenting complaint.

Diagnosis requires bilateral optic neuritis, myelitis and 2 of the following 3 criteria:

  • Spinal cord lesion involving 3 or more spinal levels
  • Initially normal MRI Brain
  • Aquaporin 4 +Ve serum antibody
145
Q

Describe Dural Ectasia, its symptoms, and causes.

A

Widening or ballooning of the dural sac surrounding the spinal cord. Usually occurs in the lumbosacral region as this is where the cerebrospinal fluid pressure is greatest.

Features:

  • Lower back pain
  • Headaches
  • weakness and numbness above and below the involved limb.
  • Incontinence
  • symptoms are exacerbated by upright posture and relieved by lying down.

Causes:

  • Marfans syndrome
  • Ehlers-danlos syndrome
  • Neurofibromatosis type 1
  • Ankylosing spondylitis
  • trauma
146
Q

Describe autonomic dyreflexia, its symptoms, and management

A

A medical emergency, most often occurs in individuals with spinal cord injuries with spinal lesions above the T6 level (although has been known to occur in patients with a lesion as low as T10. thought to be triggered by afferent stimuli below the level of the lesion.

Acute AD is a reaction of the autonomic nervous system to overstimulation. It is characterised by paroxysmal hypertension, associated with throbbing headaches, profuse sweating, nasal stuffiness, flushing of the skin above the lesion, bradycardia, confusion.

Management:

  • Catheterisation, or relief of blocked catheter
  • Manual Rectal evacuation
  • GTN Spray or oral clonadine if precipiting trigger cannot be found.
147
Q

Which cranial nerves are sensory, motor or both?

A

Some Say Marry Money But My Brother Says Big Brains Matter Most

I - Sensory
II - Sensory
III - Motor
IV - Motor
V - Both
VI - Motor
VII - Both
VIII - Sensory
IX - Both
X - Both
XI - Motor
XII - Motor
148
Q

Describe Mediation Overuse Headache, its features, and management

A

Medications overuse headache is one of the most common causes of chronic daily headache. It may affect up to 1 in 50 people

Features:

  • present for 15 days or more per month
  • developed or worsened whilst taking regular symptomatic medications
  • patients using opioids and triptans are most at risk.
  • may be psychiatric co-morbidity

Management:

  • Simple analgesics and triptans should be withdrawn abruptly (may initially worsen headaches)
  • Opioid analgesics should be gradually withdrawn.
149
Q

What is Bruns syndrome?

A

A sudden onset of severe headaches and vomiting associated to a vestibular syndrome provoked by abrupt change in head position.

It is related to an episodic obstructive hydrocephalus caused by an intraventricular mass that act like a ball-valve mechanism

150
Q

What is Cerebellar Ataxia with neuropathy and bilateral vestibular areflexia (CANVAS) syndrome?

A

An adult onset slowly progressive neurologic disorder comprised of Cerebellar Ataxia, Neuropathy, and vestibular areflexia.

Features:

  • Characteristic sign of impaired visually enhanced vestibule-ocular reflex ( a impairment of three compensatory eye movement reflects, vestibule-ocular reflex (Opposing movement of the eyes to movement on the head in order to keep focussed imaged centred), smooth pursuit (smooth movement of eyes when following an object, as opposed to saccadic) and optokinetic reflex (when following an object and it move out of the line of vision the eyes centre back on the point at which the object first appeared)
  • Non length-dependant neuropathy
151
Q

Describe Narcolepsy and its features.

A

Associated with HLA-DR2, it is associated with low levels of orexin (hypocretin), a protein which is responsible for controlling appetite and sleep patterns. Early onset of REM sleep.

Features: Typical onset in teenage years, hypersomnolence, cataplexy (sudden loss of muscle tone often triggered by emotion), sleep paralysis, vivid hallucinations on going to sleep or waking up.

Investigations:
-Multiple sleep latency EEG.

152
Q

Describe Parinaud’s Syndrome and its features

A

aka dorsal midbrain syndrome, vertical gaze palsy, and sunset sign. It is an inability to move the eyes up and down. It is caused by compression of the vertical gaze centre at the rostral interstitial nucleus of the medial longitudinal fasiculus.

Features:

  • Paralysis of upwards gaze (vertical palsy is supranuclear, so Doll’s head maneucer should elevate the eyes, but eventually all upward gaze mechanism fail
  • Pseudo-argyll robertson pupils, accomodative paresis ensures, and pupils becomes mid-dilated and show high-near dissociation.
  • convergence-retractino nystagmus (on fast up gaze the eyes pull in and the globes retract)
  • eyelid retraction (collier’s sign)
153
Q

Describe Malignant MCA syndrome

A

A condition whereby blood supply from MCA is restricted leading to a widespread infarct. This leads to cerebral oedema which causes raised ICP and may lead to herniation and/or brainstem death. Mannitol or hypertonic saline is used to manage, emergency hemicraniotomy may be needed.

154
Q

Describe Corticobasal Degeneration syndrome, and its features

A

Rare progressive neurodegenerative disease involving the cerebral cortex and the basal ganglia. It is classified as one of the Parkinson Plus syndromes. Also associated with Frontotemporal dementia and Tau pathology.

Features: Parkinsonism (asymmetric), Alien hand syndrome a characteristic feature wear a limb appears to move on its own, Apraxia, or dystopia, Progressive non-fluent Aphasia, Myoclonus

155
Q

Describe the management of neuropathic pain.

A
  • First line treatment: amitriptyline, duloxetine, gabapentin or pregabalin
  • If the first line drug foes not work try one of the other three.
  • Tramadol may be used for breakthrough pain
  • Topical Capsaicin may be used for localised neuropathic pain.
156
Q

Describe Cubital Tunnel Syndrome

A

Due to the compression of the ulnar nerve

Features:

  • Intially intermittent tingling in the 4th and 5th finger
  • may be worse when the elbow is resting on a firm surface or flexed for extended periods.
  • Later numbers in the 4th and 5th finger with associated weakness
157
Q

Describe Radial tunnel syndrome

A

Most commonly due to compression of the posterior interosseous branch of the radial nerve. It is though to be a result of overuse.

Features:

  • Symptoms are similar to lateral epicondylitis making it difficult to diagnose.
  • However, the pain tends to be around 4-5cm distal to the lateral epicondyle
  • Symptoms may be worsened by wanted the elbow and pronating the forearm.
158
Q

Describe Brown-Séquard syndrome, its features, and causes.

A

Caused by damage to one hand of the spinal cord.

Features:

  • Ipsilateral loss of proprioception
  • Contralateral loss of pain + temperature
  • Ipsilateral limb paralysis

Causes:

  • Spinal cord tumour (Menigioma)
  • Trauma (most common)
  • Ischaemia
  • Infections e.g. TB
  • Inflammation e.g. MS
  • Decompression sickness.
159
Q

What are the different features of axonal vs demyelinating neuropathy in nerve conduction studies?

A

Axonal: Decreased Amplitude but normal velocity

Demyelinating: Normal Amplitude but decreased velocity

160
Q

Describe Cavernous Sinus thrombosis, its features, and management.

A

Usually caused by spreading infection in the nose, sinuses, ears or teeth, most commonly Staph Aureus and Strep.

Features: Visual disturbance, chemosis (Conjunctival swelling), exophthalmos, headaches, cranial nerve palsy III-VI (VI most common).

Management:

  • MR Venogram most sensitive imaging
  • Broad-spectrum antibiotics
  • Anticoagulation
  • Surgery may be indicated if neurological deterioration.
161
Q

Describe Chronic Paroxysmal Hemicrania, and its features

A

Aka Sjaastad syndrome, it falls under the Trigeminal Autonomic Cephalgia category of headache.

Features: Debilitating unilateral headache affecting the area around the eye. Usually consists of multiple (often more than 5/day) severe, yet short (5-30mins), headache attacks affecting only one side of the cranium. No neurological features (unlike migraine), may be associated with redness and watering of the eye but no nausea or vomiting.

Management:

  • NSAIDs especially Indomethacin. (Thought that if not responsive to indomethacin is unlikely correct diagnosis)
  • Topiramate may have a role. OR ?Lamotrigine.
162
Q

Describe Tolosa-Hunt syndrome, its features, and management,

A

A rare disorder characterised by severe and unilateral headaches with orbital pain, along with weakness and paralysis of certain eye muscles. Thought to be due to inflammation of the areas behind the eyes (Cavernous sinus and superior orbital fissure). Diagnosis of exclusion.

Features: Usually unilateral intense sharp pain and paralysis of muscles around the eye. Other features include, double vision, fever, fatigue, vertigo, arthralgia.

Management:

  • Corticosteroids
  • Steroid-sparing immunosuppressants.
163
Q

Describe Transverse myelitis and its features

A

A rare neurological condition in which the spinal cord is inflamed.

Features: Weakness and numbers of the limbs, deficits in sensations and motor skills. Autonomic dysfunction. Exact symptoms variable and reflect the level of the affected spinal cord. Typically develop over the course of hours or days. Often associated with a sensory level.

164
Q

What are the features of a posterior communicating artery aneurysm?

A

Most common of all intracranial aneurysms presents with ipsilateral third nerve palsy up to 60% of the time.

165
Q

Categorise Antiepileptics drugs according to their enzyme inducing activity

A

Enzyme Inducing AEDs:

  • Carbamazepine
  • Phenobarbital
  • Phenytoin
  • Topiramate

Non-Enzyme Inducing AEDs:

  • Acetazolamide
  • Ethosuximide
  • Levetiracetam
  • Sodium Valproate

Lamotrigine is non-enzyme inducing but there is evidence that OCP lowers its levels.

166
Q

Describe Cerebral Autosomal Dominant Arteriopathy with Subcortical Infarcts and Leukoencephalopathy (CADASIL) and its features

A

Most common form of hereditary stroke disorder thought to be due to mutations of the Notch 3 gene on chromosome 19

Features: Recurrent migraines, TIAs or strokes, usually occurring between 40 -50yrs.

167
Q

Describe Wolfram (DIDMOAD) syndrome and its features

A

Rare Autosomal-Recessive disorder. Two genetic forms WFS1 + WFS2 (Dysfunction of CISD2 gene)

Feature: Childhood onset Diabetes Insipidus Diabetes Mellitus, Optic Atrophy and Deafness

168
Q

Describe Paraneoplastic Cerebellar Degeneration and its features.

A

A paraneoplastic syndrome associated with lung, ovarian, breast, Hodgkins lymphoma.. It is believed to be due to a autoimmune reaction target Purkinje cells. Thought to be triggered when tumour cells express proteins normally expressed in the cerebellum.

Features: Dysarthria, truncal,limb and gait ataxia, and nystagmus. Symptoms develop sub acutely and progress rapidly over a period over weeks or months to a plateau period which could last for months to years.. May involve anti-purkinje cell or anti-Yo antibodies.

169
Q

Describe Chiari Malformations and its features.

A

A structural defect in the cerebellum characterised by a downward displacement of one of both or the cerebellar tonsils through the foramen magnum.

There are four types:
-Type 1 is the most commonly observed in children, in this type the lower part of the cerebellum but not brainstem extends into the foramen magnum. It is also the only type that can be acquired

-type 2 is usually only even in children with spinal bifida it also known as classic or Arnold Chiari malformation, Involves both the cerebellum and brain stem extending into the foramen magnum.

  • Type 3 is the most serious form and involves protrusion or herniation of cerebellum and brain stem through foramen magnum
  • Type 4 involves an underdeveloped cerebellum.

Features: Headache, Dysphagia, Dizziness, Vomiting, Neck pain, Ataxia, Paraesthesia. Downbeat nystagmus

170
Q

What is MELAS syndrome?

A

Mitochondrial Encephalopathy, Lactic Acidosis, and Stroke-like episodes (MELAS), a mitochondrial cytopathy (A group also included MERRF and Leber’s Hereditary optic neuropathy).

171
Q

Describe Reflex sympathetic dystrophy, its symptoms, and management.

A

Aka Complex regional pain syndrome type 1, a disorder which manifests as extreme disproportional pain which occurs after an illness or injury thought to be due to alteration of pain perception and inflammatory molecules released from peripheral nerves leading to inappropriate cross talk between sensory and motor fibres.

Features: Pain, swelling, limited range of motion, and changes to skin and bones.

Management:

  • Physical and occupational therapy.
  • Psychological treatments
  • Neuromodulation (mirror box therapy, graded motor imagery)
  • Bisphosphonates, calcitonin, ketamine.
172
Q

What are the distinguishing features of Botulinsm that helps differentiate it from myasthenia gravis?

A

There is autonomic involvement in botulism e.g. mydriatic eyes and dry mouth.

173
Q

Describe Orbital apex syndrome, its features and causes.

A

aka Jacod syndrome, a collection of cranial nerve deficits associated with a mass lesions near the apex of the orbit of the eye. Most commonly optic nerve is involved.

Features: Most common finding is oculomotor nerve dysfunction leading to opthalmoplegia. Often accopanied by thalamic nerve dysfunction leading to hypoeasthetsia of the upper face. Optic nerve may also be involved resulting in visual impairment.

Causes: Head and neck cancer, neural tumours, haematological cancer, sarcoid, SLE, GPA, Churg-strauss, GCA, Thyroid disease, Iatrogenic, carotid aneurysm

174
Q

Describe Sturge-Weber syndrome and its types and features.

A

A rare congenital neurological and skin disorder. It is one of the phakomatoses and is often associated with port-wine stains of the face, glaucoma, seizures, mental retardation, and ipsilateral leptomengingeal angioma (cerebral malformations and tumours.

Types:

  • Type 1 include facial and leptomeningeal angiomas as well as the possibility of glaucoma or choroidal lesions, Normally only ones side of the brain is affected, this type is the most common
  • Type 2 involvement includes a facial angioma (Port wine stain) with a possibility of glaucoma developing. There is no envidence of brain involvement. Symptoms can show at any time beyond the initial diagnosis of the facial angioma. The symptoms can include glaucoma, cerebral blood flow abnormalities and headaches.
  • Type 3 has leptomengingeal angioma involvement exclusively. The facial angioma is absent and glaucoma rarely occurs. This type can only be diagnosed via brain scans.
175
Q

Describe Hereditary Neuralgic amyotrophic and its features

A

An Autosomal Dominant disorder associated in 50% of cases with a mutation of the SEPT9 gene on chromosome 17. Characterised by nerve damage, muscle atrophy, preceded by severe pain.

Features: An episodic disorder characterised by episodes generally lasting 1-6 weeks. During an episode nerves of the brachial plexus are targeted by the body as antigens and are degenerated. Leading to loss of function and pain, if phrenic nerve is involved there can be difficulty with breathing and the recurrent laryngeal nerve leads to dysphonia.

Management:
-Prednisone. Nerve regenerates after an episodes though in some cases permanent damage may occur.

176
Q

Describe Familial Amyloid Polyneuropathy and its features.

A

Aka Transthyretin amyloidosis. An Autosomal Dominant neurodegenerative disease. Characterised by systemic deposition of amyloidogenic variants of the transthyretin protein, especially in the peripheral nervous system causing a progressive sesnory and motor polyneuropathy. Caused by a mutation of the TTR gene on Chromosome 18q12.1-11.2 a replacement of valaline by methionine at potion 30 is the most common mutation.

Features: Usually manifesting between 20-40yrs. Characterised by pain, paraesthesia, muscular weakness, and autonomic dysfunction. In its terminal state the kidneys and heart are affected.

Management:
Liver transplantation.

177
Q

Describe Idiopathic Intracranial Hypertension, its features, investigations, and management.

A

Characterised by increased intracranial pressure without a detectable cause. More common in overweight can be triggered by tetracyclines.

Features: Headache, (usually worst in the mornings generalised throbbing, worsened by coughing.) N&V, visual disturbance, pulsatile tinnitus, shoulder pain.

Investigations:

  • Fundoscopy (Papiloedema
  • CT Head r/o SOL
  • LP

Management:

  • Weight loss, exercise, salt restriction
  • Acetazolamide
  • rarely neurosurgery (optic nerve sheath fenestration)
178
Q

Describe Conus Medullaris Syndrome, its features.

A

A collection of signs and symptoms associated with injury to the conus medullaris (The tapered lower end of the spinal cord occur near L1/L2).

Features: Sudden bilateral distal paresis. Urinary retention and faecal incontinence. Back pain more severe than radicular pain. Symmetrical sensory loss or perianal region. Preserved knee eras but absent ankle keys. Fasciculations.

179
Q

What are the differences between cauda equina and conus medullaris syndromes?

A

Presentation: Sudden and bilateral in conus and gradual and often unilateral in cauda.

Reflexes: Knee jerks preserved and ankle jerks hyperreflexic in conus whereas both affected in cauda and are absent.

Radicular pain: less severe in conus more in cauda

Back pain: More severe in conus less in cauda

Sensory symptoms: Bilateral perianal sensory loss, sensory dissociation occurs in conus whereas in cauda numbness is more saddle area and asymmetrical and may be unilateral. There is also loss of sensation in dermatomes of lower extremities.

Motor strength: Distal hyper-Reflex is apreasis of lower limbs. Fasciculations may be present in conus. In cauda assymteric paraplegia areflexic. Atrophy more common.

Sphincter dysfunction: Early urinary retention and faecal incontinence in conus, whereas urinary retention in cauda is a late feature

180
Q

Describe Multifocal motor neuropathy, its features, and management.

A

A progressive pure motor neuropathy often mistaken for amyotrophic lateral sclerosis. MMN only involves the lower motor neurones. Thought to be caused by autoantibodies against GM1.

Features: Usually beginning in one or both hands MMN is characterised by weakness, muscle atrophy, cramping, and often profuse fasciculations. Sensory nerves are usually unaffected. Wrist drop and foot drop are common symptoms. Cold and Hot temperatures exacerbates MMN symptoms.

Management:

  • IVIg
  • Cyclophosphamide, rituximab.
  • Prednisolone can exacerbate symptoms.
181
Q

Describe Idiopathic Brachial Plexopathy

A

Aka Parsonage-Turner syndrome

Features: abrupt onset of shoulder pain (Usually unilateral) followed by progressive neurological deficits of motor weakness and paraesthesia may be associated with recent viral illness.

182
Q

Describe X-linked adrenoleukodystrophy, its features

A

An X-linked disease that results from fatty acid build up caused by dysfunction of peroxisomal fatty acid beet oxidation which leads to damage to the myelin sheath of nerves resulting in seizures and hyperactivity. Caused by mutations in the ABCD1 gene. All patients with ALD are at risk for adrenal insufficiency.

Features: emotional instability, hyperactivity, disruptive behaviour at school. Later muscle stiffness, paraparesis and sexual dysfunction. Adrenal insufficiency.

183
Q

Describe Hemiballismus and its management

A

A movement disorder which manifests as unilateral involuntary flinging movements of the Proximal upper limbs. The lesion is in the contralateral subthalamic nucleus of Lucy’s

Management:

  • Stroke TIA prevention
  • Haloperidol Lowe dose and tirated May help
  • Tetrabenazine (A catecholamine-depleting agent) may be considered when long-term therapy is required.
184
Q

What factors increase the risk of seizure recurrence on withdrawal of AED’s?

A

Older age at diagnosis
Use of multiple antivconvulsants
History of Myoclonic or tonic-clonic seizures.
Previous abnormal imaging or EEG

185
Q

Describe Becker’s muscular dystrophy, its features

A

X-linked recessive disorder characterised by slowly progressing muscle weakness of the legs and pelvis. Caused by mutations in the dystrophin gene, similar to Duchenne muscular dystrophy but due to partial function of dystrophin is less severe and presents later.

Features: Muscular weakness (Gradually increasing difficulty walking), Upper extremity muscle weakness, Toe-walking, SOB, Pseudohypertrophy of calf muscles, cardiomyopathy, Elevated CK.

186
Q

What are the distinguishing features between Duchenne and Becker’s Muscular dystrophy?

A

Duchenne:

  • Absent/Non functional dystrophin gene
  • 1/3600 Male births incidence
  • Onset 3-5 yrs
  • Fast disease progression
  • Mean life expectancy mid 20s
  • Onset of cardiomyopathy is after skeletal progression.

Becker’s:

  • Partially functional dystrophin gene.
  • 6/100 000 male births incidence
  • Onset 12 yrs
  • Slow disease progression
  • Mean life expectancy 40s
  • Onset of cardiomyopathy May present before skeletal symptoms.
187
Q

Describes Spinal Muscular Atrophy, its features, and types.

A

A rare neuromuscular disorder characterised by loss of motor neurones and progressive muscle wasting often leading to early death. Genetic defect in the SMN1 gene.

Types:

  • SMA1: aka Werdnig-Hoffman disease, infantile SMA presenting 0-6 months with floppy limbs and respiratory impairment
  • SMA2: aka Dubowitz disease, intermediate SMA, presents at 6-18months when able to sit up but not stand, Longer life expectancy.
  • SMA3: aka Kugelberg-Welander disease, Juvenile SMA, occurs after 12 months, able to walk without support when disease occurs. Life expectancy near normal.
  • SMA4: Adult onset, 3rd decade of life with gradual proximal muscle weakness. Normal life expectancy.
188
Q

Describe Vertebral Artery Dissection and its features

A

May originate in the neck and extend to the intracranial portion of the vessel or remain isolated to either segment. Ehlers-Danilo’s and Fibromuscular dysplasia are risk factors.

Triggers: Rapid and extreme rotation of the neck, such as chiropractic manipulation or extension of the neck to have ones hairs washed, forceful coughing.

Features: Symptoms may fluctuate over minutes to hours. Symptoms consist of posterior circulation deficits (Vertigo, diplopia, dysarthria) or infarction of the cervical spinal cord from occlusion of the anterior spinal artery. Axial MRI T1 weigted images show a doub le unmentioned in the dissected vessel and may reveal areas of infarction.

189
Q

Describe Bickerstaff’s Encapahlitis and its features, and management.

A

Brainstem Encephalitis, it may be related to Gillian-Barre syndrome. And in particular the Miller-fisher variant with which is shares the opthalmoplegia and ataxia. However in miller-fisher there is often areflexia and no disturbance of conciousness but similarly there is positive anti-GQ1b antibodies. It is normally an antecedent illness following LRTI.

Features: progressive fairly symmetrical Opthalmoplegia, ataxia, disturbance of conciousness, and extensor plantar response +/- hyperreflexia.

Management:
-Steroids, immunoglobulins and plasma exchange.

190
Q

Describe Lance-Adams syndrome and its management

A

A rare condition describing post-hypoxic intention myoclonus usually following cerebral hypoxia e.g. cardiac arrest.

Management:
-Levetiracetam, clonazepam, valproate are first line treatments.

191
Q

Describe the HINTS exam

A

A bedside test used to rule out dangerous vestibular symptoms e.g. central causes.

Comprises of Head Impulse test, Nystagmus, Test of Skew.

A benign exam is defined as an ABNORMAL Head impulse test, Direction Fixed horizontal Nystagmus, and absent skew

A dangerous HINTS exam is any one of the follow

  • Normal head impulse or untestable.
  • Direction changin horizontal nystagmus
  • Skew deviation present or untestable

A dangerous HINTS results was found to be 100% sensitive and 96% specific for the presence of a central lesion when applied to a patient with acute vestibular syndrome with at least one stroke risk factor.

192
Q

Describe Subacute Degeneration of the Cord, its features and management.

A

Degeneration of the posterior and lateral columns of the spin cord as a result of Vitamin B12 defiency, Vitamin E and Copper deficiency. Most often associated with Pernicious anaemia.

Features:
Gradual and uniform onset of weakness of legs and arms, and trunk. Tingling and numbness that progressively worsens. Visual changes and confusion may occur. Bilateral spastic paresis can occur with associated pressure, vibration and touch sensory loss.

Management:
-Treat the cause i.e vitamin b12 replacement and neuro rehab,

193
Q

What is the helicopter sign?

A

A murmur sounding like a helicopter when auscultating the calves in orthostatic tremor